LSAT and Law School Admissions Forum

Get expert LSAT preparation and law school admissions advice from PowerScore Test Preparation.

 elsa10
  • Posts: 1
  • Joined: Mar 20, 2019
|
#63523
I eliminated A because it is out of scope. I eliminated B because it is irrelevant. I eliminated C because he is suggesting a correlation not a cause. I eliminated D because I think it is a mistaken reversal, because a high nutrient diet doesn't necessitate non violence, rather a poor diet correlates with violence. I chose E because it proves that the study is working because only the individuals who changed their diets have seen any improvement. Is this correct? I had a hard time between D and E, but in the end I was convinced that E is correct. Can someone go over why D is wrong?
 James Finch
PowerScore Staff
  • PowerScore Staff
  • Posts: 943
  • Joined: Sep 06, 2017
|
#63543
Hi Elsa,

The issue with (D) is that it doesn't really tell us anything; we're trying to see what causes violent behavior, so we need a control group of violent offenders who eat low-nutrient food and an experimental group that eats more nutritious food. The stimulus tells us about the experimental group and that they showed behavioral improvements, so all we lack is a control group that doesn't show behavioral improvements. (E) provides this, while (D) just says that in a survey "many" (which tells us nothing about the actual proportion) of the offenders that chose to eat high-nutrition diets were non-violent. Great, but how does this compare to the group that chose low-nutrition food? We don't know, and so it's meaningless information. We already know that violent inmates were choosing low-nutrition food, so we would assume there would be a lower proportion of high-nutrition-diet offenders that were violent. And (D) doesn't even tell us that.

Hope this clears things up!
 LearntheLSAT
  • Posts: 16
  • Joined: Sep 15, 2019
|
#68292
I'm taking the online course, and as part of our homework, I'm completing some Strengthen Q's on page 4-21 of the PS book, LW4.

I chose C originally as the stim mentions in the first observation "...the violent inmates...chose, from the food available, those items that were low in nutrients." This doesn't explicitly state "diet" nor does it exclude the low-nutrient food. From the explanations on the thread earlier, this is why they negate answer C. I'm a little perplexed on how this doesn't strengthen the argument, even just a little as it reinforces that:

Poor Nutrition (cause) -> Violent Behavior in Young Offenders (effect)

I would really appreciate some clarification :-D
 James Finch
PowerScore Staff
  • PowerScore Staff
  • Posts: 943
  • Joined: Sep 06, 2017
|
#68302
Hi LearntheLSAT,

(C) is a classic trap choice for people who haven't studied the LSAT or are reading too quickly and miss the key words. Many of the young offenders, meaning some of them, had eaten at least a single low nutrient food sometime in the days leading up to attacking someone. Laid out this way, we can see that the scope is incredibly vague ("many") and also unhelpful as to whether a low-nutrient diet, as a whole, causes violent behavior, as eating a single low-nutrient food several days before committing violent acts doesn't provide any linkage between overall diet and violence.

Hope this clears things up!
 hope
  • Posts: 84
  • Joined: Feb 13, 2018
|
#75786
Dave Killoran wrote:Hi Aba,

The big problem with (D) is the shell game being played with the terms here, the one that you referred to in your comments about violent vs nonviolent. The stimulus refers to "the violent inmates among them [young offenders]" whereas this answer choice references the entire group of "young offenders." So, whereas the stimulus talks about violent going to nonviolent, we don't know that that result is what occurs in answer choice (D), and that difference opens a big hole in this answer. There's a further issue that could be raised about the fact that the offenders selected that diet on their own (self-selecting samples are a problem), but the first issue mentioned here is enough to kill (D) for us.

Please let me know if that helps. Thanks!
Hi Dave and thank you for existing. You've really helped us students with your bibles and all else. I didn't select E because I thought that it represented a mistaken negation of the causal conclusion. It was my understanding that a mistaken negation would never be correct. Also, one of your instructors represented the causal conclusion as "good nutrition causes less violence". But the stimulus clearly represents the causal conclusion as poor nutrition causes violence. Which is it? And is it safe to deviate from the actual wording of the stimulus' causal conclusion? If I had not been so focused on the causality of this question I would have selected E. But given I've been taught to focus on the conclusion, I messed up. Can you help me? Thank you.
 hope
  • Posts: 84
  • Joined: Feb 13, 2018
|
#75787
Hi Dave and thank you for existing. You've really helped us students with your bibles and all else. I didn't select E because I thought that it represented a mistaken negation of the causal conclusion. It was my understanding that a mistaken negation would never be correct. Also, one of your instructors represented the causal conclusion as "good nutrition causes less violence". But the stimulus clearly represents the causal conclusion as poor nutrition causes violence. Which is it? And is it safe to deviate from the actual wording of the stimulus' causal conclusion? If I had not been so focused on the causality of this question I would have selected E. But given I've been taught to focus on the conclusion, I messed up. Can you help me? Thank you.
User avatar
 KelseyWoods
PowerScore Staff
  • PowerScore Staff
  • Posts: 1079
  • Joined: Jun 26, 2013
|
#75800
Hi Hope!

It sounds like you're confusing causal reasoning with conditional reasoning. Remember that these are two very different and distinct methods of reasoning--but it is certainly easy to get them to confused!

Mistaken Negations (as well as Mistaken Reversals and Contrapositives) only apply to conditional reasoning. In conditional reasoning, your sufficient condition indicates that a necessary condition must also occur and it is very important that you understand that the conditional relationship only goes in one direction so that you don't fall into the Mistaken Negation and Mistaken Reversal traps. It sounds like you understand how important the direction of that relationship is and you're already looking out for those traps--so great job!

In causal reasoning, the cause activates the effect. There's a temporal relationship (the cause has to happen before the effect) but it is not the same type of absolute unidirectional relationship that you have with conditional relationships. Rather, the cause and the effect go hand in hand. So when you have the cause, you should have the effect and when you have the effect you should have the cause. Of course, it is very difficult to prove causal relationships definitively, which is why causal conclusions are inherently flawed.

As you said, it's important to focus on the conclusion, but you have to consider the conclusion in the context of the argument rather than focusing on the conclusion to the exclusion of everything else. By reminding you to focus on the conclusion, we mean that you need to be very clear on the precise wording of the conclusion as well as clear on the premises that the author is using to support that conclusion--and often why those premises as stated don't fully support the conclusion as stated.

So in this question, the author is drawing a causal relationship between nutrition and behavior. As you point out, the conclusion tells us that there's a link between poor nutrition and violent behavior. But why does the author say that these results confirm this link? Because in the experiment, the inmates placed on a diet high in nutrients showed an improvement in behavior. So good nutrition causes violent behavior to improve; bad nutrition causes violent behavior to be worse. The cause and the effect are each along a spectrum from poor to good nutrition, and from more violent to less violent behavior. Basically the causal relationship is between nutrition and violent behavior such that poor nutrition causes more violent behavior and good nutrition causes less violent behavior.

With a causal conclusion in a Strengthen question, you are basically looking for an answer choice that does one of five things: a) eliminates an alternate cause; b) shows when the cause occurs, the effect also occurs; c) shows when the cause does not occur, the effect does not occur; d) eliminates the possibility that it is the reverse cause and effect; or e) supports the data. In this case, answer choice (E) strengthens the causal link between nutrition and violent behavior by saying when you don't have the good nutrition (the cause of improved behavior in the experiment), you don't have an improvement in violent behavior (the effect of good nutrition in the experiment). It strengthens the causal relationship by showing that the inmates on the high nutrition diet had an improvement in behavior that the inmates on the low nutrition diet did not, suggesting that the change in nutrition is what resulted in the change in behavior.

So there are two main takeaways here:

1.) Be careful not to confuse conditional and causal reasoning

2.) Focus on the conclusion within the context of the rest of the argument.

Hope this helps!

Best,
Kelsey
 hope
  • Posts: 84
  • Joined: Feb 13, 2018
|
#75831
Dave Killoran wrote:Hi Aba,

The big problem with (D) is the shell game being played with the terms here, the one that you referred to in your comments about violent vs nonviolent. The stimulus refers to "the violent inmates among them [young offenders]" whereas this answer choice references the entire group of "young offenders." So, whereas the stimulus talks about violent going to nonviolent, we don't know that that result is what occurs in answer choice (D), and that difference opens a big hole in this answer. There's a further issue that could be raised about the fact that the offenders selected that diet on their own (self-selecting samples are a problem), but the first issue mentioned here is enough to kill (D) for us.

Please let me know if that helps. Thanks!
Dave isn't answer E a mistaken negation? How could that be right?
 Jeremy Press
PowerScore Staff
  • PowerScore Staff
  • Posts: 1000
  • Joined: Jun 12, 2017
|
#75862
Hi Hope!

The relationship the author of the stimulus is arguing for is a causal one (poor nutrition being "at the root" of violent behavior means that it is a "root cause" of such behavior), not a conditional one. So we don't have to worry about mistaken negations and reversals here. Those only apply to conditional relationships.

On the other hand, if a causal relationship actually holds, then it means in the absence of the root cause (which the author has identified as poor nutrition) we should see an absence of the effect (violent behavior) as well. That "absence of cause, absence of effect" is, in essence, what the study participants in the stimulus illustrate: their better diets (i.e. an absence of poor nutrition) are associated with less violent behavior (i.e. an absence of violent behavior). Thus, the study results in the stimulus do lend some support to the conclusion that poor nutrition is a root cause of violent behavior. But answer choice E gives even more support to that conclusion, because, without the better diets, we should see that the violent behavior is still present (i.e. poor nutrition probably is at the root of the violent behavior of such inmates).

I hope this helps!

Jeremy
User avatar
 Dave Killoran
PowerScore Staff
  • PowerScore Staff
  • Posts: 5853
  • Joined: Mar 25, 2011
|
#75863
Hi Hope,

This is a good example of where you are required to know the form of reasoning you are working with, and the type of question they are asking. This is a causal argument, not a conditional one, so Mistaken Negation does not apply. Instead, we see one of the classic ways of strengthening a causal argument: "Show that when the cause does not occur, the effect does not occur."

Note also that this is a Strengthen question, not a Must question, so we aren't looking at inferences and you couldn't say a Mistaken Negation would be "wrong" just as-is even if this were conditional—it's not the right question type for that : )

Thanks!

Get the most out of your LSAT Prep Plus subscription.

Analyze and track your performance with our Testing and Analytics Package.